lichenrachel
Thanks Received: 0
Forum Guests
 
Posts: 22
Joined: July 18th, 2010
 
 
 

Q24 - When the rate of inflation

by lichenrachel Mon Aug 23, 2010 11:16 pm

I have no clue what this question is talking about... Seems like economic principles are involved. I am totally confused. Someone please help me...
User avatar
 
ManhattanPrepLSAT1
Thanks Received: 1909
Atticus Finch
Atticus Finch
 
Posts: 2851
Joined: October 07th, 2009
 
 
 

Re: Q24 - When the rate of inflation

by ManhattanPrepLSAT1 Thu Aug 26, 2010 12:20 am

Fortunately I majored in economics back in school, so to me it's not jibberish. Though I bet for many people it is - just like a discussion on the use of metaphors would be way over my head!

The question asks us to complete the argument. We need to infer from the other statements, a logical completion to the argument.

We know that the difference between the rate of return on the most profitable investment and the rate of inflation is equal to the minimum loss possible.

What we don't know is the difference between the rate of return on a particular investment and the rate of inflation.

The stimulus relates to a situation in which the loss on the investment was greater than the minimum loss. If we suffered a loss on an investment greater than the minimum loss, then we must have not invested in the most profitable investment! Best expressed in answer choice (C).

The rest of the answer choices simply hope that you don't hold the conditions fixed. Remember the conclusion said, "in such a situation." So we need to hold the conditions constant, eliminating answer choices (A) and (B).

(A) cannot be inferred.
(B) cannot be inferred.
(D) is very tempting, but does not need to be true. Remember, it's the return on the particular investment that has declined, not the minimum loss on an investment.
(E) is similar to answer choice (E) but also not necessarily true.

Let me know if you still need some more help with this one!
 
perng.yan
Thanks Received: 2
Forum Guests
 
Posts: 51
Joined: November 05th, 2010
 
 
trophy
First Responder
 

Re: PT10 S4 Q24 When the rate of inflation exceeds the rate

by perng.yan Sat Dec 04, 2010 9:22 pm

......????? hahah.... this is the one of the most confusing questions.. i feel it needs a specific background to be able to answer it...

but anyway...

i wanted to ask why (A) is wrong. haha.. i was looking at it and thought to myself.. "this cannot be the right answer because it just restates a part of the stimulus" but it was the only one that really made sense to me..

if rate of inflation exceeds the rate of return, then it will be the minimum value loss. therefore... if the value is more than that minimum.. doesn't it mean that there was a greater inflation rate???
 
rpcuhk
Thanks Received: 5
Forum Guests
 
Posts: 41
Joined: May 02nd, 2011
 
 
 

Re: Q24 - When the rate of inflation exceeds the rate

by rpcuhk Fri Sep 30, 2011 12:17 pm

To perng.yan,
for(A) to be true, it must also be true that the particular investment is the most profitable investment
 
john.o.wray
Thanks Received: 5
Vinny Gambini
Vinny Gambini
 
Posts: 13
Joined: February 08th, 2012
 
 
 

Re: Q24 - When the rate of inflation

by john.o.wray Sat Jun 09, 2012 7:03 pm

A would, in fact, explain the result, but it is not necessary. The prompt asked for what "must be true".

This is really a must be true question. C) is correct because it is the only thing that is true under all circumstances. A and D both give answers that would explain the result, but they are not necessarily true.
 
rukmini.p.reddy
Thanks Received: 0
Forum Guests
 
Posts: 1
Joined: September 01st, 2012
 
 
 

Re: Q24 - When the rate of inflation

by rukmini.p.reddy Sat Sep 01, 2012 2:46 pm

I'm still not clear on why D is wrong. If the rate of return declined, wouldn't the minimum loss on the investment increase?
 
randitect
Thanks Received: 0
Forum Guests
 
Posts: 15
Joined: November 11th, 2012
 
 
 

Re: Q24 - When the rate of inflation

by randitect Sun Nov 11, 2012 7:30 am

Is B wrong (and the explanatory power of A and D as well) because we're not talking about a series of calculations, but rather a one-time calculation??
Since the results of this one time calculation does not conform with the results necessary for the most profitable investment, C is correct?

Please let me know if my thinking is correct. Otherwise, I'm lost! I chose B...
 
tzyc
Thanks Received: 0
Atticus Finch
Atticus Finch
 
Posts: 323
Joined: May 27th, 2012
 
 
trophy
Most Thankful
 

Re: Q24 - When the rate of inflation

by tzyc Tue Mar 05, 2013 3:35 am

I still don't get what's happening in the stimulus and why the answer is (C)... :oops:
Could anyone paraphrase it please?

Thank you
 
fmuirhea
Thanks Received: 64
Forum Guests
 
Posts: 46
Joined: November 29th, 2012
 
This post thanked 4 times.
 
 

Re: Q24 - When the rate of inflation

by fmuirhea Tue Mar 05, 2013 7:20 pm

Fill-in-the-blank questions can be tough because they're not a consistent question type: sometimes they're asking for the main point, sometimes a hard inference (like here), and I've even seen them asking for a strengthener. So, you have to read carefully to determine the question task. Here, the key phrase "it must be true that" immediately precedes the blank, so this is a MBT inference question, as john.o.wray noted.

When numbers and percentages are involved, I often find it easier to add in some hypothetical numbers to think my way through the relationships. The long first sentence gives us a conditional relationship:


rate of inflation > rate of return on most profitable investment --> difference between rates is minimum % of decline


This makes sense: if inflation is outstripping the rate of return, then you're going to lose the difference. As Matt noted, the key phrase in this argument is "in such a situation." Let's invent a hypothetical situation to work with:


rate of inflation: 20%
rate of return on most profitable investment: 12%
minimum decline: 8% (i.e., 20%-12%)


This is the minimum decline because this is the best investment available. Presumably, there are other investment options, but they must all have worse rates of return (i.e., lower than 12%) - if they didn't, then 12% wouldn't be the most profitable rate of return. So, these other investments would lose you even more money!

The blank wants us to fill in the second half of another conditional relationship that is started in the second sentence with the trigger "if:"


decline of a particular investment > minimum decline --> [?]


The "minimum decline" (highlighted in blue) corresponds to the 8% decline we outlined earlier (also highlighted in blue). Let's toss in another hypothetical to account for the term "decline of a particular investment" (highlighted in red): let's try 14% (this certainly qualifies as greater than 8%, so it's valid). Working backwards, the rate of return on this investment would be 6% - i.e., it cannot be the most profitable investment available, because we already know its rate of return is 12%.

In fact, no matter what number you choose for decline of a particular investment (14%, 12%, 19%, anything else greater than 8%), since it has to be greater than 8%, we are forced to the conclusion that the investment in question was not the most profitable.

Moreover, no matter what hypothetical numbers you started off with, since the decline of a particular investment has to be greater than the minimum decline, it must be that this particular investment was not the one with the best rate of return in every hypothetical situation.
 
zen
Thanks Received: 0
Jackie Chiles
Jackie Chiles
 
Posts: 27
Joined: August 01st, 2015
 
 
 

Re: Q24 - When the rate of inflation

by zen Tue Sep 15, 2015 5:01 pm

Alright, this problem is really tough. Many of the explanation seem really complicated too(the subject matter/jargon is really throwing me off) and I still don't feel entirely clear so I'll attempt to write out my thought process.

So "Rate of Inflation" - "Rate of Return" for the most profitable investment available = the minimum loss.
Any investment that has a greater minimum loss won't be the most profitable investment available because we have established a guideline for what defines the most profitable investment available--its minimum loss. So if the min. loss is 12%, any investment that has a greater min. loss i.e., 15%, will not be the most profitable investment available.

Note: the min. loss would be greater for the non-most profitable investment available because it would have certain characteristics that would cause this such as a lower rate of return. If inflation stayed the same and the rate of return decreased, the minimum loss amount would go up because of the equation in the last paragraph. A similar effect would occur with inflation went up and the rate of return stayed the same or if the rate of return went down while inflation rose. In all these cases the difference (the result from our subtraction) would increase.

So the stimulus tells us that a particular investment(notice how it gives no indication whether we are talking about the most profitable investment available or not) has a minimum loss greater than the guideline we established. So we already know it can't be the most profitable investment available because it has a greater minimum loss than is required for the most profitable investment available!

Answer Choices-

(A)- We don't know. It could have risen, but if also could have declined and just the rate of return went down--this would explain how it declined. ELIMINATE

(B)- Eliminating this one has been tough for me. But we can eliminate it because we do not know that this investment is becoming less profitable( although it seems to that it is a likely possibility), we only know the minimum loss is greater and that this particular investment can't be the MPIA.

(D)- Two reasons why this is wrong: 1) We already know we aren't talking about the most profitable investment available, rather another investment due to this particular investment's minimum loss being greater than the baseline we established as defining the most profitable investment available; and 2) Like (A), we don't know it was the rate of return that caused the minimum loss to rise, it could have been that inflation rose; we don't know for sure one way or another( note that the greater the minimum loss, the greater the value as declined because you are losing more money! Something is more valuable if it gets you more money, if it gets you less, it is less valuable!). ELIMINATE

(E)- Similar to (D), we know we are not talking about the most profitable investment available, so we cannot conclude the MPIA (i should have used this acronym a LONG time ago :p) has changed! We are given no information about the MPIA just about a particular investment that is not the MPIA.
ELIMNATE

(C)- Correct answer. Going back to our original inference; we know this investment can't be the MPIA as it does not adhere to our min.loss stipulation/guideline. So, if it not our MOST PROFITABLE available investment, it HAS to be a less profitable investment! Bingo.

Hope this helps some! Sorry my explanation for (B) was a little weak; it's hard for me to understand right now. :evil: